2006 AMC 8 Problems/Problem 4

Revision as of 22:56, 5 September 2011 by Math Kirby (talk | contribs) (Created page with "== Problem == Initially, a spinner points west. Chenille moves it clockwise <math> 2 \dfrac{1}{4}</math> revolutions and then counterclockwise <math> 3 \dfrac{3}{4}</math> revolu...")
(diff) ← Older revision | Latest revision (diff) | Newer revision → (diff)

Problem

Initially, a spinner points west. Chenille moves it clockwise $2 \dfrac{1}{4}$ revolutions and then counterclockwise $3 \dfrac{3}{4}$ revolutions. In what direction does the spinner point after the two moves?

[asy]size(96); draw(circle((0,0),1),linewidth(1)); draw((0,0.75)--(0,1.25),linewidth(1)); draw((0,-0.75)--(0,-1.25),linewidth(1)); draw((0.75,0)--(1.25,0),linewidth(1)); draw((-0.75,0)--(-1.25,0),linewidth(1)); label("$N$",(0,1.25), N); label("$W$",(-1.25,0), W); label("$E$",(1.25,0), E); label("$S$",(0,-1.25), S); draw((0,0)--(-0.5,0),EndArrow);[/asy]

$\textbf{(A)}\ \text{north} \qquad  \textbf{(B)}\ \text{east} \qquad  \textbf{(C)}\ \text{south} \qquad  \textbf{(D)}\ \text{west} \qquad  \textbf{(E)}\ \text{northwest}$

Solution

If the spinner goes clockwise $2 \dfrac{1}{4}$ revolutions and then counterclockwise $3 \dfrac{3}{4}$ revolutions, it ultimately goes counterclockwise $1 \dfrac{1}{2}$ which brings the spinner pointing $\boxed{\textbf{(B)}\ \text{east}}$.